Diplomat: Every major war in the last 200 years has been preceded by a short, sharp increase in the acquisition of w...

weepa7 on July 25, 2015

Help

Somebody please explain why it is a and not b.

Replies
Create a free account to read and take part in forum discussions.

Already have an account? log in

Naz on August 5, 2015

Conclusion: "arms control agreements will preserve peace."

Why? "Every major war in the last 200 years has been preceded by a short, sharp increase in the acquisition of weapons by the nations that subsequently became participants in those conflicts."

Let's diagram the premise:

If there was a major war in the last 200 years, then it was preceded by a short, sharp increase in the acquisition of weapons by the nations that subsequently became participants in those conflicts.

MW ==> IAW
not IAW ==> not MW

This argument is switching the necessary and sufficient conditions of its premise and falsely interpreting it to mean that if there is an increase in acquisition of weapons, then there will certainly be a major war. However, we know this to be false. We cannot use a necessary condition to infer any other information. Further, remember that we can look to the necessary conditions of the principle rule and its contrapositive to create a possible scenario.

So, in this case, we can have "PIAW" and "not MW" together, i.e. we can have an increase in acquisition of weapons, while not having a major war.

Answer choice (A) is correct because it points out exactly what we discussed above, that the diplomat is incorrectly switching the necessary and sufficient conditions. Let's diagram (A):

We have an unless formula. Remember, we diagram A, unless B, by negating the sufficient condition and keeping the necessary condition as is. So we would diagram this example as "not A ==> B."

So, (A) would be diagrammed:

If an event of the second type occurs (an increase in acquisition of weapons), then an event of the first type (a major war) occurs.

(A): IAW ==> MW

As you can see, we have switched the variables here, but have forgotten to negate. Thus, answer choice (A) clearly points out the flaw in diplomat's argument.

Answer choice (B) is incorrect because nowhere in the argument did the diplomat say that it is because weapons are used in war that a rapid increase in the acquisition of weapons will lead to war. We have not been given any info on what happens during a war. We merely know that when a major war occurs, it is preceded by a short, sharp increase in the acquisition of weapons.

Remember, the limit to your knowledge on the LSAT is found in the boundaries of the passage. If it is not explicitly stated, then we do not know it. Thus, we cannot say that the argument reasons in this way since we have not been given any information to back this up.

Hope that helps! Please let us know if you have any other questions.

bb042745 on September 5, 2020

I am confused by the answer as much as the question. I thought we were, on these types of questions, suppposed to accept the premise and look for the logic flaw. Here, the diplomat observes that every major war over the last 200 years was preceded by an increase in accumulation of weapons. MW --> IAW. The CP not IAW --> not MW, suggests his argument is valid (not IAW == weapons control) and not MW == peace. In her answer, the instructor criticizes this observation as false and diagams it IAW --> MW, which says that when you have an increase in accumulation of weapons, there is a major war. That, clearly is NOT what the diplomat said; nor is that accurate. Can you help? Thanks